Integrals

Calculus (Integrals, Series)
Post Reply
User avatar
Tolaso J Kos
Administrator
Administrator
Posts: 867
Joined: Sat Nov 07, 2015 6:12 pm
Location: Larisa
Contact:

Integrals

#1

Post by Tolaso J Kos »

Prove that:

\( \rm{i.} \) \( \displaystyle \int_{0}^{1}\frac{\ln x}{x-1}\, dx=\sum_{n=1}^{\infty }\frac{1}{n^2}=\zeta (2)=\frac{\pi^2}{6} \).

\( \rm{ii.} \) \( \displaystyle \int_{0}^{1}\frac{x-1}{\ln x}\, dx=\ln 2 \)
Imagination is much more important than knowledge.
User avatar
Grigorios Kostakos
Founder
Founder
Posts: 461
Joined: Mon Nov 09, 2015 1:36 am
Location: Ioannina, Greece

Re: Integrals

#2

Post by Grigorios Kostakos »

\(\rm{i.}\) \begin{align*}
\displaystyle \int_{0}^{1}{\frac{\log x}{x-1}\, dx}&=\int_{0}^{1}{\log x\biggl({-\mathop{\sum}\limits_{n=0}^{\infty}x^n}\biggr) dx}\\
&=\mathop{\sum}\limits_{n=0}^{\infty}\biggl({\int_{0}^{1}{-x^n\log{x} \, dx}}\biggr)\\
&=\mathop{\sum}\limits_{n=0}^{\infty}{\frac{1}{(n+1)^2}}\\
&=\mathop{\sum}\limits_{n=1}^{\infty}{\frac{1}{n^2}}=\zeta(2)=\frac{\pi^2}{6}\,.
\end{align*}


\(\rm{ii.}\) \begin{align*}
\displaystyle \int_{0}^{1}{\frac{x-1}{\ln x}\, dx}&\mathop{=\!=\!=\!=\!=\!=\!=}\limits^{\begin{subarray}{c}
{t\,=\,-\log{x}}\\
{-{\rm{e}}^{-t}dt\,=\,dx}\\
\end{subarray}}\, -\int_{+\infty}^{0}{\frac{{\rm{e}}^{-t}-1}{-t}\,{\rm{e}}^{-t}dt}\\
&= \int_{0}^{+\infty}{\frac{{\rm{e}}^{-t}-{\rm{e}}^{-2t}}{t}\,dt}\\
&=\int_{0}^{+\infty}{\biggl({\int_{1}^{2}{{\rm{e}}^{-st}ds}}\biggr)\,dt}\\
&=\int_{1}^{2}{\biggl({\int_{0}^{+\infty}{{\rm{e}}^{-st}dt}}\biggr)\,ds}\\
&=\int_{1}^{2}{\frac{1}{s}ds}\\
&=\log 2-\log1\\
&=\log 2\,.
\end{align*}
Grigorios Kostakos
User avatar
Tolaso J Kos
Administrator
Administrator
Posts: 867
Joined: Sat Nov 07, 2015 6:12 pm
Location: Larisa
Contact:

Re: Integrals

#3

Post by Tolaso J Kos »

With the risk of flogging a dead horse, another solution for the second one.

\( \rm{ii.} \) Consider the integral: \( \displaystyle I(a)=\int_{0}^{1}\frac{x^a-1}{\ln x}\, dx \).
We differentiate with respect to \( a \) to get: \( \displaystyle \frac{\partial }{\partial a}I(a)=\int_{0}^{1}\frac{\partial }{\partial a}\frac{x^a-1}{\ln x}\, dx=\int_{0}^{1}x^a\, dx=\frac{1}{a+1} \) thus \( I(a)=\ln \left ( a+1 \right )+c \). However \(I(0)=0 \implies c=0 \) thus \( I(1)=\ln 2\).
Imagination is much more important than knowledge.
Post Reply

Create an account or sign in to join the discussion

You need to be a member in order to post a reply

Create an account

Not a member? register to join our community
Members can start their own topics & subscribe to topics
It’s free and only takes a minute

Register

Sign in

Who is online

Users browsing this forum: No registered users and 16 guests